If Neri sits immediately next to Manley, then Klosnik can sit directly between

brianbasuperez on January 29, 2022

Don't understand the answer to this one

I've tried different scenarios and can't get to arrive at the right answer

Reply
Create a free account to read and take part in forum discussions.

Already have an account? log in

Emil-Kunkin on January 31, 2022

Hi @angelicadc,

This is a tricky game and a tricky question. We have no universal rules that give us positions, and if we look at the structure, the ordering (as in 1-6) seems to matter far less than the positioning, as in M is next to N is next to L and so forth.

We are given a scenario that N is next to M. Once we get a scenario question, we should immediately try to figure out what else must be true given our new condition.

If N and M are next to each other, we know that P must be on the other side of N thanks to the first rule.p. We can diagram this as PNM or MNP. Note that since the actual order doesn't matter, PNM and MNP are the same for the purposes of this question.

Our second rule tells us that L is next to either M or N. Since N is already surrounded, then we know that L must be next to M, and there is only one spot left next to M. We can now update our diagram to PNML or LMNP.

We know that our last two to place are K and O, and it looks like they could go in either order. That is, OKPNML is as valid as KOPNML. However, the fact that we know that our diagram must be either PNML or LMNP is enough to solve this question.

We are asked which two K could go between.

A is impossible, since L and M must go next to each other.
B is impossible, since if K were directly between L and P there would be no room for O.
C is impossible since the only one that could go directly between N and O is P.
D is impossible since n and p must be next to each other so there is no room for K.
E appears possible, we could have K between P and O, in the form of OKPNML.